Research Group of Prof. Dr. M. Griebel
Institute for Numerical Simulation
maximize
next up previous contents
Next: Das Problem auf unbeschränkten Up: Existenz und Regularität Previous: Modellbildung und Hauptresultat

Das Problem auf beschränkten Zylindern

 

Zu a>0 seien

equation269

abgeschnittene Zylinder mit Länge 2a bzw. a. Wir betrachten in diesem Abschnitt das folgende Problem:

  equation272

  equation277

  equation281

  equation285

Der Gleichung (2.21) kommt folgende Bedeutung zu: Eine Lösung des Problems auf dem unendlichen Zylinder ist maximal bestimmt bis auf Translation, d.h. wenn tex2html_wrap_inline8023 eine Lösung ist, so auch tex2html_wrap_inline8025 für jedes tex2html_wrap_inline8027 . Bei der späteren Grenzwertbildung tex2html_wrap_inline8029 , d.h. beim Übergang von beschränkten Zylindern auf den unbeschränkten, muß daher die Lösung an einem Punkt fixiert werden, um als Grenzwert keine konstante Funktion zu erhalten. Die Gleichung (2.21) legt hierzu den ,,Beginn`` der Flammenfront an die Stelle tex2html_wrap_inline8031 .gif

Zuerst stellen wir fest, daß jede tex2html_wrap_inline7831 -Lösung der Differentialgleichung gemäß Satz 5.8 automatisch von der Klasse tex2html_wrap_inline7859 ist. Dies ermöglicht uns die problemlose Anwendung der Maximumprinzipien und der Monotonieaussagen aus Kapitel tex2html_wrap_inline8037 .

Das Problem auf einem endlichen Zylinder soll mit Hilfe einer verallgemeinerten Kontinuitätsmethode gelöst werden. Dazu wird zunächst eine a priori-Abschätzung bewiesen:

  LEMMA1019

BEWEIS. Da u der Differentialungleichung tex2html_wrap_inline8067 genügt, folgt aus dem Starken Maximumprinzip von E. Hopfgif: Falls u ein absolutes Minimum an einem Punkt innerhalb tex2html_wrap_inline8071 annimmt, so ist u konstant. Aufgrund der Randwerte (2.20) auf der rechten und der linken Zylinderwand kann u aber nicht konstant sein und somit kein Minimum im Innern annehmen. Für den Fall, daß u ein absolutes Minimum an einem Randpunkt tex2html_wrap_inline8079 annimmt, so besagt das Maximumprinzip ebenfalls, daß tex2html_wrap_inline8081 gilt. Aufgrund der Neumannrandwerte (2.19) muß ein solches Randminimum auf dem Dirichletrand tex2html_wrap_inline8083 liegen, so daß u>0 in tex2html_wrap_inline8071 folgt.

Falls es ein tex2html_wrap_inline8089 mit tex2html_wrap_inline8091 geben würde, so gäbe es aufgrund der Stetigkeit eine Umgebung tex2html_wrap_inline8093 mit tex2html_wrap_inline8095 . Wegen tex2html_wrap_inline8097 ließe sich dann ebenfalls das Maximumprinzip anwenden, und man erhielte einen Widerspruch. Also gilt tex2html_wrap_inline8099 auf tex2html_wrap_inline8101 .

Damit ist bereits eine tex2html_wrap_inline8103 -Schranke für u gefunden. Bevor diese zu einer tex2html_wrap_inline8107 -Schranke ausgedehnt werden kann, wird eine Schranke für c benötigt. Betrachte dazu die folgenden gewöhnlichen Differentialgleichungen für tex2html_wrap_inline8111 :

  equation1042

Dieses Randwertproblem ist eindeutig lösbargif. Mit der Bezeichnung tex2html_wrap_inline8113 ersieht man aus (2.18) und (2.23):

eqnarray1053

Hierbei haben wir uns die strenge Monotonie von u in tex2html_wrap_inline7801 -Richtung zunutze gemacht:

equation1067

sie folgt aus Satz 3.1 des Kapitels 3 über Monotonie. Die strenge Monotonie impliziert übrigens sofort 0<u<1 in tex2html_wrap_inline8121 .

tex2html_wrap_inline8123 erfüllt die Bedingungen des Starken Maximumprinzips von E. Hopfgif, also besitzt v kein inneres Maximum, es sei denn v ist konstant. Das Maximum liegt nicht auf dem Neumannrand tex2html_wrap_inline8129 , falls v nicht konstant ist, denn sonst müßte in einem solchen Punkt tex2html_wrap_inline8133 gelten, was die homogene Neumannrandbedingung tex2html_wrap_inline8135 aber ausschließt. Daher wird das Maximum auf dem Dirichletrand angenommen, also

equation1076

und somit

equation1079

Analog dazu erhält man über die Ungleichung

equation1081

eine obere Abschätzung für tex2html_wrap_inline8137 durch tex2html_wrap_inline8139 , es gilt also

  equation1085

Rechnet man tex2html_wrap_inline8141 und tex2html_wrap_inline8143 explizit aus, so erhält man

   eqnarray1090

Daraus ersieht man

equation1106

Weil u monoton wachsend in tex2html_wrap_inline7801 -Richtung ist, nimmt u den Wert tex2html_wrap_inline8151 aus der Gleichung (2.21) für tex2html_wrap_inline8031 an. Daher ergibt sich aus (2.29)

equation1113

Daraus läßt sich folgendermaßen eine Schranke für |c| herleiten (vgl. dazu auch die Abbildung):

equation1115

figure1124

Die Konstante K hängt dabei von tex2html_wrap_inline8141 und tex2html_wrap_inline8143 ab, diese Funktionen wiederum von a, tex2html_wrap_inline8165 , tex2html_wrap_inline8167 und M.

Mit Hilfe der tex2html_wrap_inline8103 -Schranke für u und der Schranke für |c| läßt sich nun aus dem Kapitel 5 über elliptische Abschätzungen eine tex2html_wrap_inline7831 -Schranke für u herleiten: Für beliebiges p>1 gibt es gemäß Satz 5.7 eine Konstante tex2html_wrap_inline8183 , die nur von n, p, tex2html_wrap_inline8189 , a, tex2html_wrap_inline8165 , tex2html_wrap_inline8167 und M abhängt, so daß gilt:

  equation1132

Wegen tex2html_wrap_inline8199 und tex2html_wrap_inline8201 ist die rechte Seite beschränkt durch tex2html_wrap_inline8203 . Mit Hilfe des Sobolewschen Einbettungssatzesgif läßt sich aus der tex2html_wrap_inline7831 -Abschätzung leicht eine tex2html_wrap_inline8107 -Abschätzung gewinnen: Wähle p>n, dann gilt

equation1141

mit einer Konstanten tex2html_wrap_inline8211 , die nur von tex2html_wrap_inline8189 , a, n und p abhängt. Somit folgt unmittelbar die gewünschte tex2html_wrap_inline8107 -Abschätzung für u:

equation1146

tex2html_wrap_inline7963

  LEMMA1163

BEWEIS. Zum Beweis dieses Lemmas wird eine modifizierte Kontinuitätsmethode verwendet. Konkret bedeutet das, daß die Lösung (u,c) ein Fixpunkt eines Operators tex2html_wrap_inline8231 sein wird, dessen Existenz über die Lösungseigenschaft des Leray-Schauderschen Abbildungsgrades garantiert wird. Um zu zeigen, daß der Abbildungsgrad von null verschieden ist, werden zwei Homotopien verwendet. Die erste ,,verbiegt`` den Operator tex2html_wrap_inline8231 zu einem leicht handhabbaren Operator tex2html_wrap_inline8235 , und die zweite diagonalisiert diesen zu einem Operator tex2html_wrap_inline8237 . Der Abbildungsgrad des Operators tex2html_wrap_inline8237 läßt sich dann leicht mit Hilfe des Produktsatzes bestimmen.

Zu tex2html_wrap_inline8241 , tex2html_wrap_inline8243 und tex2html_wrap_inline8245 seien

    eqnarray1179

Offenbar ist (u,c) eine Lösung des Problems (2.18) - (2.21), wenn (u,c) Fixpunkt von tex2html_wrap_inline8231 bzw. Nullstelle von tex2html_wrap_inline8253 ist.

Sei K die Konstante aus der a priori-Abschätzung (Lemma 2.2), mindestens jedoch so groß, daß

  equation1198

Setze

equation1208

Zeige nun, daß der Leray-Schaudersche Abbildungsgrad tex2html_wrap_inline8257 definiert ist, d.h. daß tex2html_wrap_inline8259 eine kompakte Störung der Identität ist und daß tex2html_wrap_inline8261 auf tex2html_wrap_inline8263 gilt. Beachte, daß auf B die tex2html_wrap_inline8267 -Topologie zugrunde gelegt wird.

Zum Beweis der Kompaktheit von tex2html_wrap_inline8269 muß gezeigt werden, daß tex2html_wrap_inline8271 präkompakt ist. Hierzu bedienen wir uns wieder der tex2html_wrap_inline7831 -Abschätzung aus Satz 5.7 und des Sobolewschen Einbettungssatzes:

eqnarray1220

Die tex2html_wrap_inline7823 -Norm von tex2html_wrap_inline8277 ist beschränkt, da wie mit früherer Argumentation gemäß dem Maximumprinzip tex2html_wrap_inline8279 gilt. Also ist tex2html_wrap_inline8281 gleichgradig stetig. Der Satz von Arzela-Ascoligif garantiert nun die Präkompaktheit dieser Menge, d.h.  tex2html_wrap_inline8283 ist eine kompakte Abbildung. Daraus folgt unmittelbar, daß tex2html_wrap_inline8269 kompakt ist und damit tex2html_wrap_inline8259 eine kompakte Störung der Identität ist.

Weiterhin gilt tex2html_wrap_inline8261 auf tex2html_wrap_inline8263 , denn aus tex2html_wrap_inline8293 folgt, daß (u,c) eine Lösung des Problems (2.18) - (2.21) ist. Gemäß der a priori-Abschätzung Lemma 2.2 folgt tex2html_wrap_inline8297 und tex2html_wrap_inline8299 , jedoch liegt (u,c) nur dann auf tex2html_wrap_inline8263 , wenn tex2html_wrap_inline8305 oder |c| = K+1 gilt.

Nachdem nun gezeigt ist, daß der Abbildungsgrad wohldefiniert ist, muß noch nachgewiesen werden, daß tex2html_wrap_inline8309 auch stetig ist. Dazu nehmen wir an, daß tex2html_wrap_inline8311 eine Folge ist, die gegen tex2html_wrap_inline8313 konvergiert, wobei die Konvergenz der tex2html_wrap_inline8315 wieder in der tex2html_wrap_inline8107 -Topologie zu verstehen ist. Sei tex2html_wrap_inline8319 . Mit der gewohnten tex2html_wrap_inline7831 -Abschätzung folgern wir wie in (2.35), daß die tex2html_wrap_inline8323 in der tex2html_wrap_inline7831 -Norm beschränkt sind. Weil beschränkte Teilmengen von tex2html_wrap_inline7831 schwach folgenkompaktgif sind, gibt es eine Teilfolge, die schwach gegen ein tex2html_wrap_inline8329 konvergiert. Da die tex2html_wrap_inline8323 gemäß dem Sobolewschen Einbettungssatz auch in der tex2html_wrap_inline8333 -Norm beschränkt sind, sind deren Gradienten gleichgradig stetig. Nach dem Satz von Arzela-Ascoli sind die tex2html_wrap_inline8323 präkompakt bzgl.  tex2html_wrap_inline8107 , durch nochmaligen Übergang zu einer Teilfolge erreicht man also, daß die tex2html_wrap_inline8323 in tex2html_wrap_inline8107 gegen tex2html_wrap_inline8343 konvergieren. Weil die eine Seite tex2html_wrap_inline8345 der Differentialgleichung somit in tex2html_wrap_inline8103 gegen tex2html_wrap_inline8349 konvergiert, so konvergiert auch tex2html_wrap_inline8351 in tex2html_wrap_inline8103 gegen eine Funktion tex2html_wrap_inline8355 . Da die tex2html_wrap_inline8323 in tex2html_wrap_inline7831 schwach gegen tex2html_wrap_inline8361 konvergieren, gilt tex2html_wrap_inline8363 , und tex2html_wrap_inline8343 ist Lösung der Differentialgleichung zu tex2html_wrap_inline8313 . Da die Lösung des linearen Problems eindeutig ist, muß tex2html_wrap_inline8369 gelten. Mit der gleichen Argumentation erhält man, daß jeder Häufungspunkt der ursprünglichen Folge tex2html_wrap_inline8323 die eindeutige Lösung ist, d.h. es gibt genau diesen einen Häufungspunkt, und die ganze Folge tex2html_wrap_inline8323 konvergiert gegen tex2html_wrap_inline8277 . Also sind tex2html_wrap_inline8283 und damit auch tex2html_wrap_inline8379 stetig. Aus der eindeutigen Lösbarkeit und der a priori-Abschätzung konnte also die Stetigkeit gewonnen werden.

Die Homotopieinvarianz des Abbildungsgrades liefert uns daher

equation1261

Untersuche nun tex2html_wrap_inline8235 : Die Lösung der Differentialgleichung tex2html_wrap_inline8383 von (2.38) für tex2html_wrap_inline8385 ist (analog zu den Lösungen tex2html_wrap_inline8141 und tex2html_wrap_inline8143 )

equation1268

Bemerke, daß tex2html_wrap_inline8391 nur von tex2html_wrap_inline7801 und nicht von y abhängt, die Elimination des einzigen von y abhängigen Terms tex2html_wrap_inline7799 in der Differentialgleichung führt also dazu, daß sich die partielle Differentialgleichung wie eine gewöhnliche verhält. Dies wird freilich erst durch die homogenen Neumannrandbedingungen auf tex2html_wrap_inline8401 ermöglicht. Da tex2html_wrap_inline8403 eine bijektive Abbildung tex2html_wrap_inline8405 ist, legt die Fixpunktgleichung c fest:

equation1274

Das nun feste tex2html_wrap_inline8409 erlaubt die Definition der zweiten Homotopie: Zu tex2html_wrap_inline8241 , tex2html_wrap_inline8243 und tex2html_wrap_inline8415 seien

  eqnarray1281

Wie oben kann man nachweisen, daß diese Homotopie zulässig ist und somit gilt:

equation1291

Der Operator

equation1295

ist nun diagonalisiert, d.h. die erste Komponente tex2html_wrap_inline8417 des Bildbereichs hängt nur von u ab, und die zweite Komponente tex2html_wrap_inline8421 nur von c. Dies ermöglicht die Anwendung des Produktsatzesgif:

eqnarray1301

Wegen tex2html_wrap_inline8425 besitzt die erste Abbildung im Produkt den Grad +1. Wegen (2.41) gilt tex2html_wrap_inline8429 und tex2html_wrap_inline8431 , so daß die zweite Abbildung den Grad -1 besitzt. Somit gilt

equation1310

Die Lösungseigenschaftgif des Abbildungsgrades garantiert nun die Existenz einer Nullstelle von tex2html_wrap_inline8231 und damit einer Lösung (u,c) des Problems (2.18) - (2.21). tex2html_wrap_inline7963


next up previous contents
Next: Das Problem auf unbeschränkten Up: Existenz und Regularität Previous: Modellbildung und Hauptresultat

Marcel Arndt
Tue Mar 28 09:56:06 MSZ 2000